Maximum Value

Algebra Level 4

a , b , c a,b,c are non-negative real numbers satisfying a + b + c = 1 a+b+c=1 .

The maximum value of

a b + b c + c a 2 a b c ab+bc+ca-2abc

can be written as m n \frac{m}{n} ; where m , n m,n are relatively prime. Find m + n m+n .


The answer is 34.

This section requires Javascript.
You are seeing this because something didn't load right. We suggest you, (a) try refreshing the page, (b) enabling javascript if it is disabled on your browser and, finally, (c) loading the non-javascript version of this page . We're sorry about the hassle.

3 solutions

Isaac Jiménez
Jul 18, 2014

For this problem we can use the Geometric - Arithmetic - Quadratic mean inequality.

First, we know that

a b c 3 a + b + c 3 \sqrt [ 3 ]{ abc } \le \frac { a+b+c }{ 3 } which is a b c 3 1 3 \sqrt [ 3 ]{ abc } \le \frac { 1 }{ 3 } involving a b c 1 27 abc\le \frac { 1 }{ 27 } . So, we must take the maximum value of a b c abc which is 1 27 \frac { 1 }{ 27 } so 2 a b c = 2 27 2abc=\frac { 2 }{ 27 } .

Then, we know

( a + b + c ) 2 = ( a 2 + b 2 + c 2 ) + 2 ( a b + b c + c a ) = 1 { (a+b+c) }^{ 2 }=({ a }^{ 2 }+{ b }^{ 2 }+{ c }^{ 2 })+2(ab+bc+ca)=1 . Now, 1 3 = a + b + c 3 a 2 + b 2 + c 2 3 \frac { 1 }{ 3 } =\frac { a+b+c }{ 3 } \le \sqrt { \frac { { a }^{ 2 }+{ b }^{ 2 }+{ c }^{ 2 } }{ 3 } } therefore, 1 3 a 2 + b 2 + c 2 \frac { 1 }{ 3 } \le { a }^{ 2 }+{ b }^{ 2 }+{ c }^{ 2 } . Then, 2 ( a b + b c + c a ) 2 3 2(ab+bc+ca)\le \frac { 2 }{ 3 } which is a b + b c + c a 1 3 ab+bc+ca\le \frac { 1 }{ 3 } , and as we want to take the maximum value a b + b c + c a = 1 3 ab+bc+ca=\frac { 1 }{ 3 } .

So now we replace the values in the first equation a b + b c + c a 2 a b c ab+bc+ca-2abc equivalent to 1 3 2 27 = 7 27 = m n \frac { 1 }{ 3 } -\frac { 2 }{ 27 } =\frac { 7 }{ 27 } =\frac { m }{ n } so m+n=27+7=\boxed { 34 } .

And great problem!!! :D

Isaac Jiménez - 6 years, 11 months ago

We can also use Cauchy-Schwarz Inequality to show that

(ab + bc + ca)^2 =< (a^2 + b^2 + c^2)(b^2 + c^2 + a^2)

ab + bc + ca =< (a^2 + b^2 + c^2)

therefore, again by applying AM-GM on RHS, we can find

max(ab+ bc + ca) = 1/3

Kartik Sharma - 6 years, 10 months ago

Log in to reply

I did same...

Dev Sharma - 5 years, 6 months ago
Jubayer Nirjhor
Jul 22, 2014

Let a b + b c + c a 2 a b c = X ab+bc+ca-2abc=X . By AM-GM inequality: ( 1 2 a ) ( 1 2 b ) ( 1 2 c ) ( 3 2 ( a + b + c ) 3 ) 3 = 1 27 (1-2a)(1-2b)(1-2c) \le \left(\dfrac{3-2(a+b+c)}{3}\right)^3=\dfrac{1}{27} Expanding the left side gives us: 1 2 ( a + b + c ) + 4 ( a b + b c + c a 2 a b c ) = 4 X 1 1-2(a+b+c)+4(ab+bc+ca-2abc)=4X-1 Hence: 4 X 1 1 27 X 7 27 4X-1 \le \dfrac{1}{27} ~~ \Longrightarrow ~ X \le \boxed{\dfrac{7}{27}} with equality iff 1 2 a = 1 2 b = 1 2 c a = b = c = 1 3 1-2a=1-2b=1-2c~\Longrightarrow a=b=c=\dfrac{1}{3} .

If we optimize the solution, we can take a=b=c=1/3. and it has to result with the maximum value because we have left no scope for any further increment. thus. 1/9+1/9+1/9-2/27=7/27; 7+27=34

Chitra Singh - 6 years, 10 months ago
Lorenzo Moulin
Sep 17, 2015

This problem can be answered by the postulate: "if the sum is constant, the product is maximum when all of terms are equal"

0 pending reports

×

Problem Loading...

Note Loading...

Set Loading...